Mock AIME 3 Pre 2005 Problems/Problem 13

Revision as of 10:37, 4 April 2012 by 1=2 (talk | contribs)

Problem

$13.$ Let $S$ denote the value of the sum

$\left(\frac{2}{3}\right)^{2005} \cdot \sum_{k=1}^{2005} \frac{k^2}{2^k} \cdot {2005 \choose k}$

Determine the remainder obtained when $S$ is divided by $1000$.

Solution

This problem needs a solution. If you have a solution for it, please help us out by adding it.

See Also

Mock AIME 3 Pre 2005 (Problems, Source)
Preceded by
Problem 12
Followed by
Problem 14
1 2 3 4 5 6 7 8 9 10 11 12 13 14 15